Đến nội dung

viet9a14124869 nội dung

Có 855 mục bởi viet9a14124869 (Tìm giới hạn từ 30-03-2020)



Sắp theo                Sắp xếp  

#721689 Chứng minh: (a2 + b2)2 > 8(a - b)2

Đã gửi bởi viet9a14124869 on 25-04-2019 - 20:05 trong Bất đẳng thức và cực trị

 Cho a > b và a.b = 1. Chứng minh: (a2 + b2)2  > 8(a - b)2

$\Leftrightarrow [a^2+b^2-2\sqrt{2}(a-b)].[a^2+b^2+2\sqrt{2}(a-b)]>0$ (*)

Lại có ab=1 nên :

          (*) $\Leftrightarrow (a-b-\sqrt{2})^2.(a-b+\sqrt{2})^2 \geq 0  $ 

 

 

 

...  Hình như bài toán có dấu bằng mà nhỉ ?? ... 




#720473 $x+y+z\leq \alpha. Chứng minh (1+\frac{1}{...

Đã gửi bởi viet9a14124869 on 25-02-2019 - 20:39 trong Bất đẳng thức và cực trị

Cái đề ngu nhất từng thấy ...




#712924 $(a+b+c)^3\geq 6\sqrt{3}(a-b)(b-c)(c-a)$

Đã gửi bởi viet9a14124869 on 21-07-2018 - 09:41 trong Bất đẳng thức và cực trị

Giả sử c = min {a;b;c} . Áp dụng bất đẳng thức AM-GM 

$$[6\sqrt{3}(a-b)(b-c)(c-a)]^2 \leq 108(a-b)^2.a^2.b^2=27.(a-b)^2.2ab.2ab\leq [(a-b)^2+2ab+2ab]^3=(a+b)^6\leq (a+b+c)^6$$

Từ đó suy ra điều phải chứng minh , dấu bằng xảy ra khi c=0 , $a=(2+\sqrt{3}).b$ và các hoán vị .




#712360 CMR : $3(a+b+c)\leqslant \sqrt[3]{26+a^3}+\sqrt...

Đã gửi bởi viet9a14124869 on 11-07-2018 - 18:10 trong Bất đẳng thức và cực trị

 

$$\sqrt{2}\left ( a+ b+ c \right )\left ( a+ b \right )\left ( b+ c \right )\left ( c+ a \right )\geqq 8\left ( \sqrt{a^{2}+ bc}+ \sqrt{b^{2}+ ca}+ \sqrt{c^{2}+ ab} \right )$$

Gần tương tự ( Phạm Kim Hùng ) :

$$3(a+b+c)\geq 2(\sqrt{a^2+bc}+\sqrt{b^2+ca}+\sqrt{c^2+ab})$$




#712323 Tìm $M \in d$ sao cho $2MA+MB$ nhỏ nhất

Đã gửi bởi viet9a14124869 on 10-07-2018 - 20:14 trong Hình học phẳng

Lời giải của bạn có vẻ đã ... đúng. Ta có thể áp dụng BĐT Cauchy luôn cho biểu thức cuối thì sẽ không phải khảo sát. Dĩ nhiên, lời giải muốn có kết quả đẹp, ít tính toán thì phải phụ thuộc vào m, n. Trong đề thi tỉnh VP, người ta cũng cho sẵn tọa độ các điểm và phương trình đường thẳng d. 

 

Thay số 2 bằng hằng số $k>0$ cũng vẫn giải được theo cách này.

Lời giải sẽ thế nào nếu thay điều kiện M thuộc đường thẳng d thành M thuộc 1 đường tròn ạ ?? ...




#712309 $$\left ( a^{2}+ b^{2}+ c^{2}...

Đã gửi bởi viet9a14124869 on 10-07-2018 - 15:57 trong Bất đẳng thức và cực trị

$$a^{4}\,+\, b^{4}\,+\, c^{4}\,+\, 17\,\left ( a^{2}b^{2}+ b^{2}c^{2}+ c^{2}a^{2} \right )\,\geqq\, 6\,\left ( a+ b+ c \right )\,\left ( a^{2}b+ b^{2}c+ c^{2}a \right )$$

Lời giải cụ thể :

Không mất tính tổng quát giả sử a=min{a;b;c}. Đặt b=a+x , c=a+y ($x,y\geq 0$) , bất đẳng thức đã cho tương đương : 

$a^4+(a+x)^4+(a+y)^4+17.[a^2(a+x)^2+a^2(a+y)^2+(a+x)^2(a+y)^2)]\geq 6(3a+x+y)[a^2(a+x)+(a+x)^2(a+y)+(a+y)^2)a)]$

$$\Leftrightarrow 4(x^2-xy+y^2)a^2-2(x^3+x^2y-8xy^2+y^3)a+x^4-6x^3y+11x^2y^2+y^4 \geq 0$$

Xét biệt thức delta : 

$$\Delta =4(x^3+x^2y-8xy^2+y^3)^2-16(x^2-xy+y^2)(x^4-6x^3y+11x^2y^2+y^4)$$

$$=-12(x^3-5x^2y+2xy^2+y^3)^2\leq 0$$

Do đó theo định lí về dấu của tam thức bậc 2 ta có điều phải chứng minh .

 

 

P/S: Có nhầm chỗ nào không nhỉ, anh em xem giùm  :rolleyes: :huh: :huh: ...




#712303 $$\left ( a^{2}+ b^{2}+ c^{2}...

Đã gửi bởi viet9a14124869 on 10-07-2018 - 11:33 trong Bất đẳng thức và cực trị

$$a^{4}\,+\, b^{4}\,+\, c^{4}\,+\, 17\,\left ( a^{2}b^{2}+ b^{2}c^{2}+ c^{2}a^{2} \right )\,\geqq\, 6\,\left ( a+ b+ c \right )\,\left ( a^{2}b+ b^{2}c+ c^{2}a \right )$$

Mình nghĩ cách tiếp cận tốt nhất với học sinh phổ thông cho bài toán như này là giả sử c = min {a; b; c} , đặt a=c+x ,b=c+y rồi khai triển dùng tam thức bậc 2 ...




#712302 $\sum (\dfrac{a}{b+c})^2+128\dfrac...

Đã gửi bởi viet9a14124869 on 10-07-2018 - 11:30 trong Bất đẳng thức - Cực trị

Gợi ý: Đi chứng minh bổ đề:

$$(\dfrac{a}{b+c})^2+(\dfrac{b}{a+c})^2+(\dfrac{c}{a+b})^2 \geq \dfrac{a^4+b^4+c^4}{(ab+bc+ca)^2}$$

i. Nếu một trong ba số a,b,c bằng 0 thì bổ để trở thành đẳng thức .

ii. Nếu không số nào trong ba số này bằng 0, theo bất đẳng thức Cauchy-Schwarzt : 

$$LHS \geq \frac{(a^4+b^4+c^4)^2}{\sum a^6(b+c)^2}$$

Vậy ta đi chứng minh :

 $$(a^4+b^4+c^4)(ab+bc+ca)^2\geq \sum a^6(b+c)^2$$

$$\Leftrightarrow a^2b^2c^2(a^2+b^2+c^2)+2abc(a^4b+b^4c+c^4a+ab^4+bc^4+ca^4)\geq 0$$

Bất đẳng thức trên đúng với mọi a,b,c dương , nên ta có điều phải chứng minh , dấu bằng xảy ra khi abc=0 .




#712196 $\frac{a(b+c)}{b^2+c^2}+...$

Đã gửi bởi viet9a14124869 on 08-07-2018 - 21:48 trong Bất đẳng thức và cực trị

Chứng minh rằng với mọi số thực không âm $a,b,c$ ta luôn có:

 

$$\frac{a(b+c)}{b^2+c^2}+\frac{b(c+a)}{c^2+a^2}+\frac{c(a+b)}{c^2+a^2} \geq \frac{(a+b+c)^2}{a^2+b^2+c^2}$$

Nhân hai vế bất đẳng thức với $a^2+b^2+c^2$ thì bất đẳng thức cần chứng minh tương đương 

 

$$[\frac{a^3(b+c)}{b^2+c^2}+a(b+c)]+[\frac{b^3(c+a)}{c^2+a^2}+b(c+a)]+[\frac{c^3(a+b)}{a^2+b^2}+c(a+b)]\geq (a+b+c)^2$$ 

$$\Leftrightarrow \frac{a^3(b+c)}{b^2+c^2}+\frac{b^3(c+a))}{c^2+a^2}+\frac{c^3(a+b)}{a^2+b^2}\geq a^2+b^2+c^2$$

$$\Leftrightarrow \sum \frac{ab(a^2+ab+b^2+c^2)}{(a^2+c^2)(b^2+c^2)}.(a-b)^2 \geq 0$$ 

Bất đẳng thức này hiển nhiên đúng do a,b,c không âm . Hoàn tất chứng minh .




#708199 Cho (O;R) đường thẳng d cắt (O) tại C và D

Đã gửi bởi viet9a14124869 on 13-05-2018 - 05:49 trong Hình học

c,

Theo tớ thì

Gọi E là giao điểm của AB với OM 

ME.MO=MA^2=MC.MD

nên dễ dàng cm OECD là tứ giác nội tiếp

$\Rightarrow \widehat{CEM}=\widehat{ODC}=\widehat{OCD}=\widehat{OED}$

$\Rightarrow \widehat{CEA}=\widehat{DEA}$

hay EA là phân giac cua goc CED 

=> AB đi qua diem F là diem chinh giua cung CD cua duong tron ngoai tiep tu giac CIOD co dinh

Điểm F này cũng chính là điểm F ở lời giải của bạn vkhoa ...




#707976 $P=\frac{2x}{\sqrt{(x^2+1)^3}}+...

Đã gửi bởi viet9a14124869 on 09-05-2018 - 19:54 trong Bất đẳng thức và cực trị

Đây mới chỉ là bấm máy tính xấp xỉ thôi :D, chứ cái cực trị nó khá lẻ

Ý mình hỏi bạn có thể trình bày phương pháp bấm máy kia không ??




#707974 $P=\frac{2x}{\sqrt{(x^2+1)^3}}+...

Đã gửi bởi viet9a14124869 on 09-05-2018 - 19:49 trong Bất đẳng thức và cực trị

Hàm số đạt cực trị khi $x,y,z$ xấp xỉ $(1;0.4;\frac{8}{3})$, Max xấp xỉ $1.4124$

Làm thế nào ra được như vậy ??




#707541 $\sqrt{5a+4}+\sqrt{5b+4}+\sqrt{5...

Đã gửi bởi viet9a14124869 on 02-05-2018 - 21:05 trong Bất đẳng thức và cực trị

Cho a,b,c không âm và a+b+c=1

Tìm max $\sqrt{5a+4}+\sqrt{5b+4}+\sqrt{5c+4}$

Áp dụng bất đẳng thức Cauchy

 $$\sqrt{5a+4}+\sqrt{5b+4}+\sqrt{5c+4}\leq \sqrt{3(5a+5b+5c+12)}=\sqrt{51}$$

Dấu bằng xảy ra khi a=b=c=$\frac{1}{3}$

 

P/S : Bài này còn tìm được min nữa =)) :

Áp dụng  bất đẳng thức Cauchy-Schwarzt

$$\sqrt{5a+4}=\sqrt{(9a+4b+4c)(a+b+c)}\geq 3a+2b+2c$$

Xây dựng các bất đẳng thức tương tự rồi cộng lại , ta có GTNN bằng 7 .

Dấu bằng xảy ra chẳng hạn như a=1,b=c=0 .




#707401 MA+MB+MC max

Đã gửi bởi viet9a14124869 on 30-04-2018 - 17:04 trong Hình học

Trên tia đối của tia MA lấy N sao cho MN=MB => MA+MB=MN+MA=AN

Gọi I là điểm chính giữa của cung BC

Trên tia đối IA lấy K sao cho IK=IB=>IK=IB=IA

=> I là tâm (AKB)

Ta có $\widehat{AKB}=\frac{1}{2}\widehat{AIB}=\frac{1}{2}\widehat{AMB}=\widehat{ANB}$

=> A,N,K,B thuộc (I;IA)

=>AN$\leq$AK

Dấu bằng xảy ra khi M trùng I

.......

Nên làm bài cẩn thận ,tôi thấy có nhiều điểm chưa thỏa ( như mấy chỗ bôi đỏ )

Với lại bạn có chắc không ?? Có thật là N luôn thuộc (I;IA) không ?? Hãy thử xem xét khi M thuộc cung AC không chứa B . :huh: 




#707288 CM $abc+2 \geq ab+bc+ca$

Đã gửi bởi viet9a14124869 on 29-04-2018 - 13:47 trong Bất đẳng thức và cực trị

 Cho a,b,c là các số dương thỏa $a^2+b^2+c^2+abc=4$

CM $abc+2 \geq ab+bc+ca$

Lời giải :

Theo bất đẳng thức Cauchy :

$$4=a^2+b^2+c^2+abc\geq2ab+c^2+abc\Rightarrow (2+c)(2-c-ab)\geq 0\Leftrightarrow 2\geq c+ab$$

Vậy ta chỉ cần chứng minh :

 $$abc+c+ab\geq ab+bc+ca\Leftrightarrow c(a-1)(b-1)\geq 0$$

 

Điều này đúng với mọi a,b thỏa mãn a-1,b-1 cùng dấu . Vậy bài toán được chứng minh xong .

Dấu bằng xảy ra với $(a;b;c)\in {(1;1;1),(0;\sqrt{2};\sqrt{2})}$ và các hoán vị của nó .




#707283 [TOPIC] ÔN THI BẤT ĐẲNG THỨC $\boxed{\text{THPT CHUYÊN}}$...

Đã gửi bởi viet9a14124869 on 29-04-2018 - 12:24 trong Tài liệu - Đề thi

Bạn yeutoan89 sửa lại bài đi nhé, gõ tex có vấn đề , với lại bạn quên chưa đánh số thứ tự bài kìa 

 

Bài toán số 110 : Giả sử rằng a,b,c là các số thực dương thỏa mãn a+b+c=1 . Chứng minh rằng : 

$$\sqrt[3]{\frac{1}{a}-4b}+\sqrt[3]{\frac{1}{b}-4c}+\sqrt[3]{\frac{1}{c}-4a}\leq \sqrt[3]{\frac{5}{3}} .(\frac{1}{15abc}+\frac{6}{5})$$ 

 

P/S : Sửa đề rồi nha ai vô chém thử đi .....

                                                                                                                                               ____ Nguyễn Đức Việt____




#707262 [TOPIC] ÔN THI PHƯƠNG TRÌNH THPT CHUYÊN 2018 - 2019

Đã gửi bởi viet9a14124869 on 29-04-2018 - 10:02 trong Tài liệu - Đề thi

Bài 41:GHPT

$\left\{\begin{matrix} x\sqrt{y^{2}+6}+y\sqrt{x^{2}+3}=7xy\\ x\sqrt{x^{2}+3}+y\sqrt{y^{2}+6}=x^{2}+y^{2}+2 \end{matrix}\right.$

Xét thấy với x hoặc y bằng 0 thì hệ vô nghiệm

Hệ phương trình tương đương : $\left\{\begin{matrix} \frac{\sqrt{x^2+3}}{x}+\frac{\sqrt{y^2+6}}{y}=7 & \\ \frac{3x}{x+\sqrt{x^2+3}}+\frac{6y}{y+\sqrt{y^2+6}}=2 & \end{matrix}\right.\Leftrightarrow \left\{\begin{matrix} a+b=7 & \\ \frac{3}{1+a}+\frac{6}{1+b}=2 & \end{matrix}\right.$

Trong đó $\left\{\begin{matrix} a=\frac{\sqrt{x^2+3}}{x} & \\ b=\frac{\sqrt{y^2+6}}{y} & \end{matrix}\right.$

Rút a=7-b rồi thay vào phương trình dưới , ta có : 

$$\frac{3}{1+(7-b)}+\frac{6}{1+b}=2$$

$$\Leftrightarrow (b-5)(2b-7)=0$$

Đến đây xét 2 trường hợp là ra , mọi người thử tiếp nhé  ^_^




#707255 Bất đẳng thức

Đã gửi bởi viet9a14124869 on 29-04-2018 - 09:47 trong Bất đẳng thức và cực trị

Cho a,b,c là các số thực thõa mãn a2+b2+c2=27 . Tìm min a3+b3+c3

Lời giải : 

 

 

Theo đề ra , ta có $a,b,c\in[-3\sqrt{3};3\sqrt{3}]$

Do đó $a^2(a+3\sqrt{3})+b^2(b+3\sqrt{3})+c^2(c+3\sqrt{3})\geq 0\Leftrightarrow a^3+b^3+c^3\geq -3\sqrt{3}(a^2+b^2+c^2)=-81\sqrt{3}$

Vậy giá trị nhỏ nhất của biểu thức là $-81\sqrt{3}$ , dấu bằng xảy ra với các bộ hoán vị của $(0; 0; -3\sqrt{3})$.




#707245 [TOPIC] ÔN THI BẤT ĐẲNG THỨC $\boxed{\text{THPT CHUYÊN}}$...

Đã gửi bởi viet9a14124869 on 29-04-2018 - 08:38 trong Tài liệu - Đề thi

Bài toán số 108 : Cho a,b,c là các số thực dương thỏa mãn a+b+c=1 . Với q=ab+bc+ca , chứng minh rằng : 

$$\frac{a}{(1-a)^2}+\frac{b}{(1-b)^2}+\frac{c}{(1-c)^2}\geq \frac{1}{2q(1-q)}$$

                                                                                                                                                              ____ Nguyễn Đức Việt ____

 

P/S : Yếu quá :P ....




#707237 [TOPIC] ÔN THI BẤT ĐẲNG THỨC $\boxed{\text{THPT CHUYÊN}}$...

Đã gửi bởi viet9a14124869 on 29-04-2018 - 07:43 trong Tài liệu - Đề thi

Anh viet9a14124869 lời giải bài này đâu cần phức tạp quá vậy ạ... 

Ta có:

$\frac{4}{a+b+2}+\frac{4a}{2a+b+1}+\frac{4b}{2b+a+1}=\frac{4}{(a+1)+(b+1)}+\frac{4a}{(a+b)+(a+1)}+\frac{4b}{(a+b)+b+1}\leq \frac{1}{a+1}+\frac{1}{b+1}+\frac{a}{a+b}+\frac{a}{a+1}+\frac{b}{a+b}+\frac{b}{b+1}=3$

Anh quên mất ^^ đi ngủ mới nghĩ ra cái này  :lol:  :lol: .....




#707211 [TOPIC] ÔN THI BẤT ĐẲNG THỨC $\boxed{\text{THPT CHUYÊN}}$...

Đã gửi bởi viet9a14124869 on 28-04-2018 - 21:21 trong Tài liệu - Đề thi

105. Cho $a\,,b\,\,> 0$. Chứng minh rằng:

 

$\frac{1}{2 + a + b} + \frac{a}{2a + b + 1} + \frac{b}{2b + a + 1} \,\, \leqq \frac{3}{4}$

Lời giải trên hay quá , rất ấn tượng =))

Bất đẳng thức cần chứng minh tương đương : 

$$(1-\frac{2a}{2a+b+1})+(1-\frac{2b}{2b+a+1})\geq \frac{2}{a+b+2}+\frac{1}{2}$$

$$\Leftrightarrow \frac{a+1}{2b+a+1}+\frac{b+1}{2a+b+1}\geq \frac{2}{a+b+2}+\frac{1}{2}$$

Áp dụng bất đẳng thức Cauchy-Schwarzt và Cauchy ta có : 

$$VT\geq \frac{[(a+1)+(b+1)]^2}{[(a+1)(2b+a+1)+(b+1)(2a+b+1)]}=\frac{(a+b+2)^2}{(a^2+4ab+b^2)+4(a+b)+2}\geq \frac{(a+b+2)^2}{\frac{3(a+b)^2}{2}+4(a+b)+2}$$

Đến đây đơn giản rồi , đặt t=a+b ( t > 0) thì ta quy về chứng minh :

$$\frac{(t+2)^2}{\frac{3t^2}{2}+4t+2}\geq \frac{2}{t+2}+\frac{1}{2}$$

$$\Leftrightarrow(t-2)^2(t+2)\geq 0$$

Điều này hiển nhiên đúng ,vậy bài toán được chứng minh.

Dấu bằng xảy ra khi a=b=1 .




#707178 $\frac{1}{a+2}+\frac{1}{b+2...

Đã gửi bởi viet9a14124869 on 28-04-2018 - 13:09 trong Bất đẳng thức và cực trị

Cho các số dương $a, b,c$ thỏa mãn $abc=1$. Chứng minh rằng: 

$\frac{1}{a+2}+\frac{1}{b+2}+\frac{1}{c+2}\leq 1$

- Sáng tác -

Bài này cũ rồi , không nhớ là của ai nữa  :icon6: 

Lời giải :

Bất đẳng thức cần chứng minh tương đương với:

$$\frac{a}{a+2}+\frac{b}{b+2}+\frac{c}{c+2}\geq 1$$

Áp dụng bất đẳng thức Cauchy-Schwarzt ,ta có : 

$$VT\geq \frac{(\sqrt{a}+\sqrt{b}+\sqrt{c})^2}{a+b+c+6}=\frac{(a+b+c)+2(\sqrt{ab}+\sqrt{bc}+\sqrt{ca})}{a+b+c+6}\geq \frac{a+b+c+6\sqrt[3]{abc}}{a+b+c+6}=1$$

 

Bài toán được chứng minh xong , dấu bằng xảy ra khi a=b=c=1 .




#707175 [TOPIC] ÔN THI BẤT ĐẲNG THỨC $\boxed{\text{THPT CHUYÊN}}$...

Đã gửi bởi viet9a14124869 on 28-04-2018 - 13:01 trong Tài liệu - Đề thi

Bài 102 Cho $a,b,c,d$ là các số thực dương thỏa mãn $a+b+c+d=1$. Chứng minh rằng
$$\dfrac {a^ 2+b^ 2+c^ 2 +d } {(a+b+c) ^3} +\dfrac {b^ 2+c^ 2+d^ 2 +a} {(b +c+d) ^3} +\dfrac {c^ 2+d^ 2+a^ 2 +b } {( c+d+a) ^3} +\dfrac {d^ 2+a^ 2+b^ 2 +c } {( d+a+b) ^3} > 4$$

(Olympic toán St Petersburg năm 2018 - Lớp 9)

Ta có thể  làm mạnh nó lên và chứng minh vế trái $\geq \frac{112}{27}$

Bài này mình sẽ nói qua ý thôi : 

Áp dụng bất đẳng thức AM-GM , ta có đánh giá :

$$\frac{a^2+b^2+c^2+d}{(a+b+c)^3)}\geq \frac{\frac{(a+b+c)^2)}{3}+d}{(a+b+c)^3)}=\frac{d^2+d+1}{3(1-d)^3)}\geq \frac{144d-8}{27}$$

$$\Leftrightarrow  (4d-1)^2(9d^2-23d+17)\geq 0$$ 

Điều này hiển nhiên đúng . Thiết lập các bất đẳng thức tương tự rồi cộng lại ta sẽ có điều phải chứng minh .




#707166 $\frac{1}{a^2+2}+\frac{1}{b...

Đã gửi bởi viet9a14124869 on 28-04-2018 - 10:56 trong Bất đẳng thức và cực trị

Thực ra lúc đầu định làm như vầy, nhưng thấy bài của Việt tưởng chặt quá nên đổi qua dồn biến :D :D

@Việt: về cách chọn hệ số, mình thường làm như sau, không biết tác giả làm kiểu gì:

Xây dựng 2 bđt cơ sở:

$\frac{1}{{{a^2} + 2}} \ge m\left( {a - 1} \right) + \frac{1}{3}\\ \frac{1}{{{a^2} + 2}} \ge na + \frac{1}{2}$

Từ đây ta thu được $m=\frac{-1}{6}$

P/s: tất nhiên nó chỉ work với mấy bài yếu, chặt hơn thì mệt lắm :D

Không biết tác giả tìm ra kiểu gì

Chắc là đoán dấu bằng (0;0;1) nên tác giả muốn làm xuất hiện đại lượng chứa tích a(a-1) ...




#707155 $\frac{1}{a^2+2}+\frac{1}{b...

Đã gửi bởi viet9a14124869 on 28-04-2018 - 06:01 trong Bất đẳng thức và cực trị

Bài này em tưởng mọi người sẽ chê dễ vì lời giải của em đơn giản lắm  :icon6:  :icon6:  :icon6:

Lời giải: 

Ta đi chứng minh: $\frac{1}{a^2+2}\geq \frac{-a}{6}+\frac{1}{2}\Leftrightarrow a(a-1)(a-2)\geq 0$ (luôn đúng)

Suy ra $\sum \frac{1}{a^2+2}\geq \frac{-(a+b+c)}{6}+\frac{3}{2}=\frac{4}{3}$

Best =3 ... Mình chỉ có chút kiến thức về UCT thoii, nhưng làm sao tìm được hệ số như này ??